This question just doesn't seem to click for me any which way I look at it... I don't see how answer choice (A) is the correct answer. If someone has any insight as to how to better approach this question, be my guest! Thanks!
http://7sage.com/ ...
http://7sage.com/lsat_explanations/lsat-june-2007-section-3-question-15/
Hey, could anyone please help me understand how answer choice B does not weaken the argument? if there were more people with more than 6 months treatment responding then the ...
Can somebody please point me in the directions of games that are similar to this one? Thanks!
https://7sage.com/lsat_explanations/lsat-77-section-3-game-4/
Hi!
Though I watched JY's video explanation on this question,
I cannot still figure out what is a significant difference between an answer choice (A) and (C).
I thought "cultural relationships" in (A) can entail the meaning of how ...
I'm still confused as to how and why Jon's explanation in the video is used to express answer choice A is correct and answer choice C is wrong. I'm not following his logic in the video. Can somebody please explain this? Also there was a comment below the ...
https://7sage.com/lsat_explanations/lsat-56-section-3-question-21
I understand Jon's explanation on this question, but I was tricked by the question's word choice "revival" in the answer choice (D).
Doesn't "revival" imply that the ballroom ...
Even though the question stem doesn't explicitly say "which one of the following could be a *COMPLETE AND ACCURATE* list of people selected" like what most of other questions do, do I still need to automatically assume that the list needs to be complete? ...
Damn, PT52 has some pretty tough LR sections, and even after a retake, I missed many of the same question again (like this one). I don't see how answer A weakens the argument nor how B doesn't.
I'm redoing some questions that I marked when I first went through the ciriculum, and I came across this tricky one. I fully see why answer D is correct, but I can't figure out what makes B incorrect. Doesn't answer B deny an alternate cause?